PT 57, LR1, Q6 Forum

Prepare for the LSAT or discuss it with others in this forum.
Post Reply
lsatextreme

Silver
Posts: 531
Joined: Thu Sep 24, 2009 2:18 am

PT 57, LR1, Q6

Post by lsatextreme » Thu Oct 07, 2010 11:06 pm

*EDIT: It's LR2, NOT LR1!! sorry!!!

I feel kind of dumb having an issue with a question 6, but it must be done. I get how A is weaken it, but why wouldn't D weaken it as well? Or is it that it wouldn't MOST weaken it?

Is it because the stimulus narrows it to minimum-wage jobs so an answer choice discussing workers who earn more than minimum wage is out of scope? Even then I feel like it can weaken the claim that unemployment will increase due to increase in minimum wage due to mininum wage jobs going away...

EastBay Jose

New
Posts: 3
Joined: Sat Jun 28, 2008 2:15 am

Re: PT 57, LR1, Q6

Post by EastBay Jose » Fri Oct 08, 2010 12:45 am

Choice D says that most workers now earn more than the current minimum wage (MW).
Even if this were true, "most" just means more than half, so it would still be possible that 49.999% of workers worked for MW, and a rise in the MW would affect a big proportion of workers.

But, even if there were a Choice F:
"98% of workers now earn more than the current minimum wage."
This still wouldn't weaken the argument, because it would still be possible that a lot of that 98% earned less than the new MW, once it was raised. If that were so, the raise to the MW would raise their pay, too.

And even if there were a Choice G:
"98% of workers now earn substantially more than the new, higher minimum wage."
This still wouldn't weaken the argument, because that would just mean that the scenario the Economist predicts would affect only a very small proportion of total workers, but the Economist never claimed that there would be a dramatic loss of jobs, just "an increase in unemployment," i.e. some loss of jobs.

Post Reply

Return to “LSAT Prep and Discussion Forum”